Difference between revisions of "2020 AIME II Problems/Problem 15"

(Solution)
(Solution 2 (Official MAA))
Line 50: Line 50:
 
Hence, by the Parallelogram Law,
 
Hence, by the Parallelogram Law,
 
<cmath> TM^2 + XY^2 = 2(TX^2 + TY^2) = 2(1143-XY^2).</cmath> But <math>TM^2 = TB^2 - BM^2 = 16^2 - 11^2 = 135</math>. Therefore <cmath>XY^2 = \frac13(2 \cdot 1143-135) = 717.</cmath>
 
<cmath> TM^2 + XY^2 = 2(TX^2 + TY^2) = 2(1143-XY^2).</cmath> But <math>TM^2 = TB^2 - BM^2 = 16^2 - 11^2 = 135</math>. Therefore <cmath>XY^2 = \frac13(2 \cdot 1143-135) = 717.</cmath>
 +
 +
==Solution 3 (No parallelogram Law)==
 +
Let <math>H</math> be the orthocenter of <math>\triangle AXY</math>.
 +
<color=#ff0>Lemma 1: <math>H</math> is the midpoint of <math>AC</math>.</color>
 +
Proof: Let <math>H'</math> be the midpoint of <math>AC</math>, and observe that <math>XBH'T</math> and <math>TH'CY</math> are cyclical. Define <math>H'Y \cap BA=E</math> and <math>H'X \cap AC=F</math>, then note that:
 +
<cmath>\angle H'BT=\angle H'CT=\angle H'XT=\angle H'YT=\angle A.</cmath>
 +
That implies that <math>\angle H'XB=\angle H'YC=90^\circ-\angle A</math>, <math>\angle CH'Y=\angle EH'B=90^\circ-\angle B</math>, and <math>\angle BH'Y=\angle FH'C=90^\circ-\angle C</math>. Thus <math>YH'\perp AX</math> and <math>XH' \perp AY</math>; <math>H'</math> is indeed the same as <math>H</math>, and we have proved lemma 1.
 +
 +
Since <math>AXTY</math> is cyclical, <math>\angle XTY=\angle XHY</math> and this implies that <math>XHYT</math> is a paralelogram.
 +
By the Law of Cosines:
 +
<cmath>XY^2=XT^2+TY^2+2(XT)(TY)\cdot cos(\angle A)</cmath>
 +
<cmath>XY^2=XH^2+HY^2+2(XH)(HY) \cdot cos(\angle A)</cmath>
 +
<cmath>HT^2=HX^2+XT^2-2(HX)(XT) \cdot cos(\angle A)</cmath>
 +
<cmath>HT^2=HY^2+YT^2-2(HY)(YT) \cdot cos(\angle A).</cmath>
 +
We add all these equations to get:
 +
<cmath>HT^2+XY^2=2(HT^2+TY^2).</cmath>
 +
We now use the conditions of the problem: <math>BH=HC=11</math> and <math>BT=TC=16</math>. Note that <math>HT \perp BC</math>, so by the Pythagorean Theorem, it follows that <math>HT^2=135</math>. We were also given that <math>XT^2+TY^2=1143-XY^2</math>, into which we substitute to get
 +
<cmath>135+XY^2=2286-2 \cdot XY^2 \implies 3 \cdot XY^2=2151.</cmath>
 +
Therefore, <math>XY^2=\boxed{717}</math>. ~ MathLuis
  
 
==See Also==
 
==See Also==

Revision as of 12:31, 15 April 2021

Problem

Let $\triangle ABC$ be an acute scalene triangle with circumcircle $\omega$. The tangents to $\omega$ at $B$ and $C$ intersect at $T$. Let $X$ and $Y$ be the projections of $T$ onto lines $AB$ and $AC$, respectively. Suppose $BT = CT = 16$, $BC = 22$, and $TX^2 + TY^2 + XY^2 = 1143$. Find $XY^2$.

Solution

Assume $O$ to be the center of triangle $ABC$, $OT$ cross $BC$ at $M$, link $XM$, $YM$. Let $P$ be the middle point of $BT$ and $Q$ be the middle point of $CT$, so we have $MT=3\sqrt{15}$. Since $\angle A=\angle CBT=\angle BCT$, we have $\cos A=\frac{11}{16}$. Notice that $\angle XTY=180^{\circ}-A$, so $\cos XYT=-\cos A$, and this gives us $1143-2XY^2=\frac{-11}{8}XT\cdot YT$. Since $TM$ is perpendicular to $BC$, $BXTM$ and $CYTM$ cocycle (respectively), so $\theta_1=\angle ABC=\angle MTX$ and $\theta_2=\angle ACB=\angle YTM$. So $\angle XPM=2\theta_1$, so \[\frac{\frac{XM}{2}}{XP}=\sin \theta_1\], which yields $XM=2XP\sin \theta_1=BT(=CT)\sin \theta_1=TY.$ So same we have $YM=XT$. Apply Ptolemy theorem in $BXTM$ we have $16TY=11TX+3\sqrt{15}BX$, and use Pythagoras theorem we have $BX^2+XT^2=16^2$. Same in $YTMC$ and triangle $CYT$ we have $16TX=11TY+3\sqrt{15}CY$ and $CY^2+YT^2=16^2$. Solve this for $XT$ and $TY$ and submit into the equation about $\cos XYT$, we can obtain the result $XY^2=\boxed{717}$.

(Notice that $MXTY$ is a parallelogram, which is an important theorem in Olympiad, and there are some other ways of computation under this observation.)

-Fanyuchen20020715

Solution 2 (Official MAA)

Let $M$ denote the midpoint of $\overline{BC}$. The critical claim is that $M$ is the orthocenter of $\triangle AXY$, which has the circle with diameter $\overline{AT}$ as its circumcircle. To see this, note that because $\angle BXT = \angle BMT = 90^\circ$, the quadrilateral $MBXT$ is cyclic, it follows that \[\angle MXA = \angle MXB = \angle MTB = 90^\circ - \angle TBM = 90^\circ - \angle A,\] implying that $\overline{MX} \perp \overline{AC}$. Similarly, $\overline{MY} \perp \overline{AB}$. In particular, $MXTY$ is a parallelogram. [asy] defaultpen(fontsize(8pt)); unitsize(0.8cm);  pair A = (0,0);  pair B = (-1.26,-4.43); pair C = (-1.26+3.89, -4.43); pair M = (B+C)/2;  pair O = circumcenter(A,B,C);  pair T = (0.68, -6.49); pair X = foot(T,A,B);  pair Y = foot(T,A,C); path omega = circumcircle(A,B,C); real rad = circumradius(A,B,C);    filldraw(A--B--C--cycle, rgb(0.98,0.81,0.69)); label("$\omega$", O + rad*dir(45), SW); filldraw(T--Y--M--X--cycle, rgb(173/255,216/255,230/255)); draw(M--T);  draw(X--Y); draw(B--T--C); draw(A--X--Y--cycle); draw(omega); dot("$X$", X, W);  dot("$Y$", Y, E); dot("$O$", O, W); dot("$T$", T, S);  dot("$A$", A, N);  dot("$B$", B, W);  dot("$C$", C, E);  dot("$M$", M, N);   [/asy] Hence, by the Parallelogram Law, \[TM^2 + XY^2 = 2(TX^2 + TY^2) = 2(1143-XY^2).\] But $TM^2 = TB^2 - BM^2 = 16^2 - 11^2 = 135$. Therefore \[XY^2 = \frac13(2 \cdot 1143-135) = 717.\]

Solution 3 (No parallelogram Law)

Let $H$ be the orthocenter of $\triangle AXY$. <color=#ff0>Lemma 1: $H$ is the midpoint of $AC$.</color> Proof: Let $H'$ be the midpoint of $AC$, and observe that $XBH'T$ and $TH'CY$ are cyclical. Define $H'Y \cap BA=E$ and $H'X \cap AC=F$, then note that: \[\angle H'BT=\angle H'CT=\angle H'XT=\angle H'YT=\angle A.\] That implies that $\angle H'XB=\angle H'YC=90^\circ-\angle A$, $\angle CH'Y=\angle EH'B=90^\circ-\angle B$, and $\angle BH'Y=\angle FH'C=90^\circ-\angle C$. Thus $YH'\perp AX$ and $XH' \perp AY$; $H'$ is indeed the same as $H$, and we have proved lemma 1.

Since $AXTY$ is cyclical, $\angle XTY=\angle XHY$ and this implies that $XHYT$ is a paralelogram. By the Law of Cosines: \[XY^2=XT^2+TY^2+2(XT)(TY)\cdot cos(\angle A)\] \[XY^2=XH^2+HY^2+2(XH)(HY) \cdot cos(\angle A)\] \[HT^2=HX^2+XT^2-2(HX)(XT) \cdot cos(\angle A)\] \[HT^2=HY^2+YT^2-2(HY)(YT) \cdot cos(\angle A).\] We add all these equations to get: \[HT^2+XY^2=2(HT^2+TY^2).\] We now use the conditions of the problem: $BH=HC=11$ and $BT=TC=16$. Note that $HT \perp BC$, so by the Pythagorean Theorem, it follows that $HT^2=135$. We were also given that $XT^2+TY^2=1143-XY^2$, into which we substitute to get \[135+XY^2=2286-2 \cdot XY^2 \implies 3 \cdot XY^2=2151.\] Therefore, $XY^2=\boxed{717}$. ~ MathLuis

See Also

2020 AIME II (ProblemsAnswer KeyResources)
Preceded by
Problem 14
Followed by
Last Problem
1 2 3 4 5 6 7 8 9 10 11 12 13 14 15
All AIME Problems and Solutions

The problems on this page are copyrighted by the Mathematical Association of America's American Mathematics Competitions. AMC logo.png